Q) Inert gas (4.03.30a)

by admin on April 7, 2010 · 18 comments

in Exam Questions

Question #30 from the April 2003 (AM) patent bar exam is in the Prometric database.

30. A patent application includes the following Claim 1:
Claim 1. A method of making an electrical device comprising the steps of:
(i) heating a base made of carbon to a first temperature in the range of 1875°C to 1925°C;
(ii) passing a first gas over said heated base, said first gas comprising a mixture of hydrogen, SiCl4, phosphorus, and methane, whereby said first gas decomposes over said heated base and thereby forms a first deposited layer of silicon, phosphorus and carbon on said heated base;
(iii) heating said base having said deposited layer to a second temperature of approximately 1620°C; and
(iv) passing a second gas over said base heated to said second temperature, said second gas consisting of a mixture of hydrogen, SiCl4, AlCl3, and methane, whereby said second gas decomposes over said heated base to form a second deposit layer adjacent said first layer, said second layer comprising silicon, aluminum and carbon.
Assuming proper support in the specification, which of the following claims, if presented in the same application, is a proper claim in accordance with the USPTO rules and the procedures set forth in the MPEP?
(A) Claim 2. The method of claim 1, wherein said first temperature is in the range of 1800°C to 2000°C.
(B) Claim 3. The method of claim 1, wherein said first gas further comprises an inert gas.
(C) Claim 4. The method of claim 1, wherein said second gas further comprises Argon.
(D) Claim 5. The method of claim 1, wherein said first gas is an inert gas such as Argon.
(E) Claim 6. The method of claim 1, wherein said second gas consists of a mixture of hydrogen, SiCl4 and AlCl3 only.

ANSWER: (B) is the most correct answer. 37 CFR § 1.75(c). Answers (A) and (E) are incorrect because they improperly seek to broaden the parent claim. 37 CFR § 1.75(c). Answer (A) broadens the range by going below the stated limit. Answer (E) broadens by trying to remove a recited component of the second gas, and covering subject matter that is not covered by the parent claim. Answer (C) is incorrect because claim 1 uses the close ended claim term “consists” in connection with the second gas, which precludes the addition of further components to the second gas in claim 4. Answer (D) is incorrect because the use of the exemplary language “such as” is improper is improper under 35 U.S.C. § 112, second paragraph, and because it is inconsistent with claim 1. See MPEP § 2173.05(d).

1 OverworkkedNo Gravatar April 28, 2011 at 3:35 pm

Repeat or close variant on 4/27/2011 MPEP E8R8

2 ohsoobviousNo Gravatar May 25, 2011 at 5:31 pm

got this one on 5/18/11

3 AnonNo Gravatar October 12, 2011 at 12:12 am

Close variant on 10/11/11

4 MattNo Gravatar November 14, 2011 at 3:55 pm

This is flawed. There are no claims 5 & 6.

5 KerstinNo Gravatar December 12, 2011 at 12:02 am

Matt-The question says to assume support for the later claims, so I don’t think it’s flawed.

6 KerstinNo Gravatar December 12, 2011 at 12:03 am

Following up to my above comment- there are no claims 2, 3, and 4. The question is fine I think.

7 MNo Gravatar January 4, 2012 at 6:53 pm

I am having trouble distinguishing answer B with answer C. The explanation seems to imply that a claim using the term “Consist” is close-ended while “Comprise” isn’t. Meaning that one can’t add to the first gas because “consist” was used, but one can add to the second gas because “comprise” was used.

Is this the correct way of analyzing this question?

Thanks.

8 MNo Gravatar January 4, 2012 at 6:58 pm

I mixed up my comment, it should say:

I am having trouble distinguishing answer B with answer C. The explanation seems to imply that a claim using the term “Consist” is close-ended while “Comprise” isn’t. Meaning that one can add to the first gas because “comprise” was used, but one can’t add to the second gas because “consist” was used.

Is this the correct way of analyzing this question?

Thanks.

9 mimiNo Gravatar September 13, 2012 at 4:31 am

M,
Claim 4 has all limitations of claim 1.
Members of the second gas only consist of the mixture of hydrogen, SiCl4, AlCl3, and methane. No more.

Thanks!

10 maalvaNo Gravatar January 4, 2012 at 10:29 pm

yes you are correct.

11 RemandedNo Gravatar January 20, 2012 at 8:17 pm

Saw this one 20 January 2012.

12 IndiJonesNo Gravatar June 30, 2012 at 1:58 am

got this 6/29/12

13 speedplaneNo Gravatar May 15, 2013 at 6:27 am

Maybe I’m over thinking this, but an inert gas is one which does not undergo chemical reactions. Accordingly, (B) is wrong because it would require an inert gas to “decompose[] over said heated base” and raise a 112 paragraph 1 rejection.

14 ButNo Gravatar May 15, 2013 at 8:31 am

The inert gas is to ensure control over the stoichiometry of the deposItion layers. It is valid

15 speedplaneNo Gravatar May 15, 2013 at 9:03 pm

So an inert gas can decompose?

16 iyerNo Gravatar July 30, 2014 at 4:50 am

this q appeared in a test recently

17 iyerNo Gravatar October 1, 2014 at 5:36 am

Verbatim appeared last week.I got it rt too

18 AaronNo Gravatar October 5, 2014 at 5:14 pm

I got this question in mid-August. Not verbatim, but fact pattern was really similar.

Previous post:

Next post: